Merge remote branch 'public/master'
[course.git] / latex / problems / Serway_and_Jewett_4 / equation27.07.tex
1 \begin{problem}
2 \emph{BONUS PROBLEM}.  
3 Derive the Equation 27.8, which gives the average intensity on a
4 screen far from a single slit relative to the maximum intensity
5 $I_\text{max}$ at $\theta=0$.
6 \begin{equation}
7   I_\text{avg} = I_\text{max} \cos^2\p({\frac{\pi d \sin(\theta)}{\lambda}})
8 \end{equation}
9
10 \emph{HINT}.  Remember from Chapter 24 that for plane waves
11 \begin{equation}
12   I = \frac{1}{2\mu_0 c} E_\text{max}^2
13 \end{equation}
14 where the electric field is perpendicular to the direction of
15 propogation.  Assume the screen is far enough away that the waves
16 emanating from the slits can be treated as plane waves.
17 \end{problem} % combines the phase difference from Equation 27.7 with vector
18 % addition for the Electric field amplitudes.
19
20 \begin{solution}
21 From the path-length argument we've used in Problems 2 and 3, we know
22 the phase difference between the light from each slit will be
23 \begin{equation}
24   \Delta \phi = \frac{2\pi}{\lambda} d \sin(\theta)
25 \end{equation}
26
27 Drawing a phasor diagram for the Electric field, we have
28 \begin{center}
29 \begin{asy}
30 import Mechanics;
31 import ElectroMag;
32
33 real u = 1cm;
34 transform t=scale(u);
35
36 real E = 1;
37 pair P = (0,0);
38
39 Vector Et = EField(t*P, u*E, 0, L="$E_t$");
40 Vector Eb = EField(t*P, u*E, 140, L="$E_b$");
41 Angle a = Angle(Et.pTip(), Et.center, Eb.pTip(), Et.mag/2, L="$\Delta \phi$");
42
43 Et.draw();
44 Eb.draw();
45 a.draw();
46 dot(P);
47 \end{asy}
48 \end{center}
49 \begin{equation}
50   \Delta \phi = \frac{2\pi}{\lambda} d \sin(\theta)
51 \end{equation}
52 The maximum electric field is thus given by
53 \begin{equation}
54   E_\text{max} = 2 E_0 \cos\p({\frac{\Delta\phi}{2}})
55 \end{equation}
56 where $E_0 = E_t = E_b$.
57 The intensity is then given by
58 \begin{align}
59   I &= \frac{1}{2\mu_0 c} E_\text{max}^2 \\
60     &= \frac{1}{2\mu_0 c} 4 E_0^2 \cos^2\p({\frac{\Delta\phi}{2}}) \\
61     &= I_\text{max} \cos^2\p({\frac{\Delta\phi}{2}}) \;,
62 \end{align}
63 where we made the substitution $I_\text{max} = I(\Delta\phi=0)$.
64 Plugging in for $\Delta\phi$,
65 \begin{equation}
66   I = I_\text{max} \cos^2\p({\frac{\pi d \sin(\theta)}{\lambda}}) \;,
67 \end{equation}
68 which is what we set out to show.
69 \end{solution}